Frage:
Warum lehnen Menschen einige einfache Quantenmodelle kategorisch ab?
G. 't Hooft
2012-08-15 14:35:55 UTC
view on stackexchange narkive permalink

Deterministische Modelle. Klärung der Frage:

Das Problem bei diesen Blogs ist, dass die Leute dazu neigen, sich gegenseitig anzuschreien. (Ich gebe zu, ich habe mich angesteckt und es ist schwierig, die elektronische Stimme nicht zu erheben.) Ich möchte meine Frage ohne ein Gefolge von Polemiken stellen.

Meine jüngsten Artikel wurden mit Skepsis aufgenommen. Damit habe ich kein Problem. Was mich stört, ist die allgemeine Reaktion, dass sie "falsch" sind. Meine Frage ist wie folgt zusammengefasst:

Hat eine dieser Personen die Arbeit tatsächlich gelesen und kann mir jemand sagen, wo ein Fehler gemacht wurde?

Nun die Details. Ich kann nicht anders, als von der Interpretation "vieler Welten" oder den "Pilotwellen" von Bohm-de Broglie angewidert zu sein, und selbst die Vorstellung, dass die Quantenwelt nicht lokal sein muss, ist schwer zu kaufen. Ich möchte wissen, was wirklich los ist, und um einige Ideen zu bekommen, konstruiere ich einige Modelle mit unterschiedlichem Grad an Raffinesse. Diese Modelle sind natürlich "falsch" in dem Sinne, dass sie nicht die reale Welt beschreiben, sie erzeugen nicht das Standardmodell, aber man kann sich vorstellen, von solch einfachen Modellen auszugehen und immer kompliziertere Details hinzuzufügen, um sie realistischer aussehen zu lassen Natürlich in verschiedenen Stadien.

Natürlich weiß ich, was die Schwierigkeiten sind, wenn man versucht, QM mit Determinismus zu untermauern. Einfache probabilistische Theorien scheitern wesentlich. Eine oder mehrere der üblichen Annahmen, die in einer solchen deterministischen Theorie getroffen werden, müssen wahrscheinlich aufgegeben werden. Ich bin mir dessen voll bewusst. Andererseits scheint unsere Welt äußerst logisch und natürlich zu sein.

Deshalb habe ich beschlossen, meine Untersuchung am anderen Ende zu beginnen. Nehmen Sie Annahmen an, die später sicherlich geändert werden müssen; Machen Sie einige einfache Modelle, vergleichen Sie diese mit dem, was wir über die reale Welt wissen, und ändern Sie dann die Annahmen nach Belieben.

Die No-Go-Theoreme sagen uns, dass ein einfaches zellulares Automatenmodell wahrscheinlich nicht funktioniert. Eine Möglichkeit, sie zu "ändern", bestand darin, Informationsverlust einzuführen. Auf den ersten Blick würde mich das noch weiter vom QM entfernen, aber wenn Sie etwas genauer hinschauen, stellen Sie fest, dass man immer noch einen Hilbert-Raum einführen kann, aber er wird viel kleiner und kann holographisch werden, was wir vielleicht tatsächlich tun wollen. Wenn Sie dann feststellen, dass der Informationsverlust eine Zuordnung vom deterministischen Modell zu QM-Zuständen grundsätzlich nicht lokal macht - während die Physik selbst lokal bleibt -, wird die Idee möglicherweise attraktiver.

Nun ist das Problem damit dass man wieder zu große Annahmen macht und die Mathematik ziemlich kompliziert und unattraktiv ist. Also ging ich zurück zu einem reversiblen, lokalen, deterministischen Automaten und fragte: Inwieweit ähnelt dies dem QM und wo geht es schief? Mit dem Gedanken, dass wir die Annahmen ändern, vielleicht Informationsverlust hinzufügen, in ein expandierendes Universum einbauen, aber alles, was später kommt; Zuerst möchte ich wissen, was schief geht.

Und hier ist die Überraschung: In gewissem Sinne geht nichts schief. Sie müssen lediglich davon ausgehen, dass wir Quantenzustände verwenden, auch wenn die Evolutionsgesetze selbst deterministisch sind. Die Wahrscheinlichkeitsverteilungen sind also durch Quantenamplituden gegeben. Der Punkt ist, dass bei der Beschreibung der Abbildung zwischen dem deterministischen System und dem Quantensystem viel Freiheit besteht. Wenn Sie sich einen periodischen Modus des deterministischen Systems ansehen, können Sie einen gemeinsamen Beitrag zur Energie für alle Zustände in diesem Modus definieren. Dies führt eine große Anzahl beliebiger Konstanten ein, sodass wir viel Freiheit erhalten.

Wenn ich diese Freiheit nutze, habe ich einige Modelle, die ich zufällig interessant finde. Ausgehend von deterministischen Systemen lande ich bei Quantensystemen. Ich meine echte Quantensysteme, keine dieser hässlichen Erfindungen. Andererseits sind sie noch weit vom Standardmodell oder von allem anderen entfernt, das anständige, wechselwirkende Partikel zeigt.

Außer der Stringtheorie. Ist das Modell, das ich erstellt habe, ein Gegenbeispiel, das zeigt, dass das, was mir jeder sagt, dass grundlegendes QM mit Determinismus unvereinbar ist, falsch ist? Nein, das glaube ich nicht. Die Idee war, dass ich irgendwo meine Annahmen ändern muss, aber vielleicht müssen auch die üblichen Annahmen, die in den No-Go-Theoremen getroffen wurden, berücksichtigt werden.

Ich persönlich denke, die Leute lehnen " Superdeterminismus" zu schnell ab. Ich lehne "Verschwörung" ab, aber das könnte nicht dasselbe sein. Der Superdeterminismus besagt einfach, dass Sie Ihre Meinung (über die zu messende Komponente eines Spins) nicht durch "freien Willen" ändern können, ohne auch die deterministischen Modi Ihrer Welt in der fernen Vergangenheit zu modifizieren. Es ist offensichtlich wahr in einer deterministischen Welt, und vielleicht ist dies eine wesentliche Tatsache, die berücksichtigt werden muss. Es bedeutet nicht "Verschwörung".

Hat jemand eine gute oder bessere Vorstellung von diesem Ansatz, ohne Namen zu nennen? Warum sind einige von Ihnen so stark davon überzeugt, dass es "falsch" ist? Trete ich auf die religiösen Gefühle von jemandem? Ich hoffe nicht.

Referenzen:

"Die Quantenmechanik diskreter Systeme mit der kanonischen Standardquantenmechanik in Beziehung setzen", arXiv: 1204.4926 [quant-ph] ;

"Dualität zwischen einem deterministischen zellulären Automaten und einer bosonischen Quantenfeldtheorie in $ 1 + 1 $ -Dimensionen", arXiv: 1205.4107 [quant-ph];

"Diskretion und Determinismus in Superstrings", arXiv: 1207.3612 [hep-th].


Weitere Reaktionen auf die gegebenen Antworten. (Das Schreiben als "Kommentar" ist fehlgeschlagen, das Schreiben als "Antwort" hat zu Einwänden geführt. Ich werde versuchen, die "Antwort" zu löschen, die ich dort nicht hätte ablegen sollen ...)

Erstens: Vielen Dank für die ausführlichen Antworten.

Mir ist klar, dass meine Frage philosophische Fragen aufwirft. diese sind interessant und wichtig, aber nicht mein Hauptanliegen. Ich möchte wissen, warum ich beim Aufbau meines Modells kein technisches Problem finde. Ich bin geschmeichelt über den Eindruck, dass meine Theorien so "einfach" zu konstruieren waren. Ich habe meine Präsentation zwar so transparent wie möglich gestaltet, aber es war nicht einfach. Es gibt viele tote Gassen, und nicht alle Modelle funktionieren gleich gut. Zum Beispiel kann der harmonische Oszillator auf einen einfachen periodischen Automaten abgebildet werden, aber dann trifft man auf technische Details: Der Hamiltonianer eines periodischen Systems scheint oben und unten unbegrenzt zu sein, während der harmonische Oszillator einen Grundzustand hat. Der zeitumkehrbare zellulare Automat (CA), der aus zwei Schritten $ A $ und $ B $ besteht, wobei sowohl $ A $ als auch $ B $ als Exponent physikalisch vernünftiger Hamiltonianer geschrieben werden können, ist selbst viel schwieriger auszudrücken als eine Hamiltonsche Theorie, weil die BCH-Reihe nicht konvergiert. Auch explizite $ 3 + 1 $ dimensionale QFT-Modelle widersetzten sich meinen Versuchen, sie als zellulare Automaten umzuschreiben. Aus diesem Grund war ich überrascht, dass der Superstring anscheinend so gut funktioniert, aber selbst hier mussten einige Tricks erfunden werden, um dies zu erreichen.

@RonMaimon. Ich wiederhole hier, was ich in einem Kommentar gesagt habe, nur weil dort die Beschränkung auf 600 Zeichen meinen Text zu sehr verzerrt hat. Sie haben das Problem in früheren Beiträgen gut dargelegt: In einer CA kann die "ontische" Wellenfunktion des Universums nur in bestimmten Modi der CA vorliegen. Dies bedeutet, dass sich das Universum nur in den Zuständen $ \ psi_1, \ \ psi_2, \ ... $ befinden kann, die die Eigenschaft $ \ langle \ psi_i \, | \, \ psi_j \ rangle = \ delta_ {ij} $ haben Die Quantenwelt, die wir beschreiben möchten, erlaubt viel mehr Zustände, die überhaupt nicht orthonormal zueinander sind. Wie könnten diese Zustände jemals entstehen? Ich fasse mit Entschuldigung für die Wiederholung zusammen:

  • Wir denken normalerweise, dass der Hilbert-Raum trennbar ist, dh in jedem infinitesimalen Volumenelement dieser Welt gibt es einen Hilbert-Raum und den gesamten Hilbert-Raum ist das Produkt all dieser.
  • Normalerweise nehmen wir an, dass jeder der Zustände in diesem gemeinsamen Hilbert-Raum einen "ontischen" Zustand des Universums darstellen kann.
  • Ich denke, das könnte sein nicht wahr sein. Die ontischen Zustände des Universums können eine viel kleinere Klasse von Zuständen bilden $ \ psi_i $; In Bezug auf CA-Zustände müssen sie eine orthonormale Menge bilden. In Bezug auf "Standard Model" (SM) -Zustände ist diese orthonormale Menge nicht trennbar, und deshalb denken wir lokal, dass wir nicht nur die Basiselemente, sondern auch alle Überlagerungen haben. Der orthonormale Satz kann dann einfach wieder auf die CA-Zustände abgebildet werden.

Ich denke nicht, dass wir über eine nicht denumerierbare Anzahl von Zuständen sprechen müssen, aber die Anzahl von CA-Zuständen ist extrem groß. Kurz gesagt: Das mathematische System ermöglicht uns die Auswahl: Nehmen Sie alle CA-Zustände, dann ist die orthonormale Menge groß genug, um alle möglichen Universen zu beschreiben, oder wählen Sie die viel kleinere Menge von SM-Zuständen, dann benötigen Sie auch viele überlagerte Zustände, um das Universum zu beschreiben . Der Übergang von einer Beschreibung zur anderen ist im mathematischen Sinne natürlich und reibungslos.

Ich vermute, dass man auf diese Weise sehen kann, wie eine Beschreibung, die auf CA-Ebene nicht quantenmechanisch ist (nur "klassische" Wahrscheinlichkeiten zulässt), uns "allmählich" dazu zwingen kann, Quantenamplituden zu akzeptieren, wenn wir uns größeren Entfernungsskalen zuwenden und beschränken uns nur auf viel niedrigere Energieniveaus. Sie sehen, in Worten, all dies mag krumm und vage klingen, aber in meinen Modellen denke ich, dass ich gezwungen bin, so zu denken, indem ich einfach die Ausdrücke betrachte: In Bezug auf die SM-Staaten könnte ich leicht entscheiden, alle zu akzeptieren Quantenamplituden, aber wenn ich mich der CA-Basis zuwende, entdecke ich, dass Überlagerungen überflüssig sind; Sie können durch klassische Wahrscheinlichkeiten ersetzt werden, ohne die Physik zu verändern, da in der CA die Phasenfaktoren in den Überlagerungen niemals beobachtbar werden.

@Ron Ich verstehe, dass Sie versuchen, etwas zu tun sonst. Mir ist nicht klar, ob Sie $ \ delta \ rho $ als Wellenfunktion interpretieren möchten. (Ich mache mir keine Sorgen über das Fehlen von $ \ mathrm {i} $, solange das Minuszeichen zulässig ist.) Meine Theorie ist viel direkter; Ich verwende die ursprüngliche "Quanten" -Beschreibung nur mit herkömmlichen Wellenfunktionen und herkömmlichen Wahrscheinlichkeiten.


(Neu seit Sonntag, 20. August 2012)

Es gibt ein Problem mit meiner Argumentation. (Ich korrigiere einige Aussagen, die ich zuvor hier gemacht hatte). Ich muss mit zwei Arten von Zuständen arbeiten: 1: Die Schablonenzustände, die überall dort verwendet werden, wo Sie Quantenmechanik betreiben, ermöglichen jede Art von Überlagerung; und 2: die ontischen Zustände, die Menge von Zuständen, die die Grundlage der CA bilden. Die ontischen Zustände $ | n \ rangle $ sind alle orthonormal: $ \ langle n | m \ rangle = \ delta_ {nm} $, daher sind keine Überlagerungen für sie zulässig (es sei denn, Sie möchten natürlich einen Vorlagenzustand erstellen). Man kann sich dann die Frage stellen: Wie kann es sein, dass wir (denken wir) überlagerte Zustände in Experimenten sehen? Sehen Experimente nicht nur ontische Zustände?

Meine Antwort war schon immer: Wen interessiert dieses Problem? Verwenden Sie einfach die Regeln von QM. Verwenden Sie die Vorlagen, um eine beliebige Berechnung durchzuführen, Ihren Status $ | \ psi \ rangle $ zu berechnen und dann zu beachten, dass sich die CA-Wahrscheinlichkeiten $ \ rho_n = | \ langle n | \ psi \ rangle | ^ 2 $ genau so entwickeln Wahrscheinlichkeiten sollten es tun.

Das funktioniert, aber die Frage bleibt unbeantwortet, und aus irgendeinem Grund sind meine Freunde auf dieser Diskussionsseite darüber verärgert.

Also fing ich an zu überlegen darüber. Ich kam zu dem Schluss, dass die Schablonenzustände verwendet werden können, um die ontischen Zustände zu beschreiben, aber dies bedeutet, dass sie irgendwo entlang der Linie auf eine orthonormale Menge reduziert werden müssen. Wie kommt es dazu? Wie kann es insbesondere sein, dass Experimente stark darauf hindeuten, dass Überlagerungen eine äußerst wichtige Rolle spielen, während diese meiner Theorie nach irgendwie so lauten, dass sie nicht ontisch sind?

Wenn ich mir die mathematischen Ausdrücke anschaue, denke ich jetzt, dass die Orthonormalität durch "Superdeterminismus" in Kombination mit Vakuumschwankungen wiederhergestellt wird. Das, was wir Vakuumzustand nennen, $ | \ Emptyset \ Rangle $, ist kein ontologischer Zustand, sondern eine Überlagerung vieler, vielleicht aller CA-Zustände. Die Phasen können so gewählt werden, dass sie beliebig sind, aber es ist sinnvoll, sie als $ + 1 $ für das Vakuum zu wählen. Dies ist eigentlich eine gute Möglichkeit, Phasen zu definieren: Alle anderen Phasen, die Sie für Nicht-Vakuum-Zustände einführen könnten, haben jetzt eine bestimmte Bedeutung.

Die Zustände, die wir normalerweise in einem Experiment berücksichtigen, sind normalerweise orthogonal zum Vakuum. Wenn wir sagen, dass wir Experimente mit zwei Zuständen durchführen können, $ A $ und $ B $, die nicht orthonormal zueinander sind, bedeutet dies, dass dies Schablonenzustände sind; Es ist einfach, solche Zustände zu konstruieren und zu berechnen, wie sie sich entwickeln. Es ist jedoch sicher anzunehmen, dass sich die ontologischen Zustände $ | n \ rangle $ mit dem nicht verschwindenden inneren Produkt mit $ A $ tatsächlich von den Zuständen $ | m \ rangle $ unterscheiden müssen, die in $ B $ auftreten. so dass trotz der Vorlage $ \ langle A | B \ rangle = 0 $. Dies liegt daran, dass sich das Universum niemals genau wiederholt. Meine physikalische Interpretation davon ist "Superdeterminismus": Wenn Alice (oder Bob) in einem EPR- oder Bell-Experiment ihre Meinung darüber ändert, was zu messen ist, arbeitet sie (er) mit Zuständen $ m $, die sich alle von allen unterscheiden gibt $ n $ an, das zuvor verwendet wurde. In den Vorlagenzuständen muss lediglich mindestens eine Änderung in einem der physischen Zustände an einer anderen Stelle im Universum angenommen werden. Der Widerspruch verschwindet dann.

Die Rolle von Vakuumschwankungen ist auch unvermeidlich, wenn man den Zerfall eines instabilen Partikels betrachtet.

Ich denke, es gibt kein Problem mit den obigen Argumenten, aber einige Leute finden es schwierig, das zu akzeptieren Die Arbeit ihres Geistes kann sich überhaupt auf Vakuumschwankungen auswirken, oder umgekehrt, dass Vakuumschwankungen ihren Geist beeinflussen können. Der "freie Wille" eines Beobachters ist gefährdet; Das wird den Leuten nicht gefallen.

Aber am beunruhigendsten wäre, dass dieses Argument implizieren würde, dass das, was meine Freunde seit vielen Jahrzehnten in Harvard und an anderen Orten unterrichten, tatsächlich falsch ist. Ich möchte bescheiden bleiben; Ich finde das störend.

Eine überarbeitete Version meines neuesten Papiers wurde jetzt an arXiv gesendet (wird wahrscheinlich ab Montag oder Dienstag verfügbar sein). Danke an alle. Meine Schlussfolgerung hat sich nicht geändert, aber ich habe jetzt genauere Argumente bezüglich der Ungleichungen von Bell und was Vakuumschwankungen mit ihnen anstellen können.

verwandte http://physics.stackexchange.com/q/34165/3229
Ich denke, das Problem ist, dass der Superdeterminismus in Ihrem Modell nicht intuitiv erklärt wird. Zumindest scheint dies der wiederkehrende Einwand zu sein. Wie ich Ihnen bereits sagte, sind viele Menschen mit Many Worlds zufrieden, weil sie dann ein "klares" Bild davon haben, was vor sich geht. Außerdem scheint es lokal und deterministisch zu sein. Wenn also jemand mit einer "tieferen" Theorie kommt, verursacht das offensichtlich viele Kopfschmerzen. Denn wie Sie sagen, sagt Ihr Modell im Moment keine der bekannten Physik voraus oder gibt Ihnen keine. Es funktioniert also nicht als gutes mentales Bild, um Quantenmechanik zu betreiben.
Als experimenteller Physiker bin ich offen für theoretische Möglichkeiten, bin mir der Geschichte der Physik und der Umkehrungen darin bewusst, von denen ich einige aus erster Hand erlebt habe. Ich bin voreingenommen gegenüber einer Realität, die darauf basiert, dass Zustände Null oder Eins sind (oder eine beliebige Ganzzahl). Das liegt wahrscheinlich daran, dass ich 1967, als wir an unserem Institut unseren ersten Computer bekamen und anfingen, mit Monte-Carlo-Veranstaltungen zu spielen, ernsthafte Meinungsverschiedenheiten mit Informatikern hatte, die behaupteten: Wir können alles mit Computern beschreiben, ohne dass jetzt CERN-Experimente erforderlich sind !! Menschen, die nicht einverstanden sind
Fortsetzung: Nach allgemeinen Grundsätzen muss jeder einen ähnlichen Hintergrund haben. Nur diejenigen, die sich mit der spezifischen Mathematik auskennen, können wirklich zur Diskussion beitragen, und ich lese mit Interesse die Kontroverse. Eine zweite Tendenz meinerseits ist, dass ich gesehen habe, wie Diskretion aus Kontinuum entsteht, aber kein intuitives Gefühl dafür habe, wie Kontinuum aus Diskretion entstehen kann.
Ich bin kein Physiker, aber ich bin gespannt, ob es einen Zusammenhang zwischen Ihrer Theorie und Belavkins Ansatz (Ereignismechanik) gibt, zum Beispiel: http://arxiv.org/abs/quant-ph/0512187http://arxiv.org/ abs / quant-ph / 0512188http: //arxiv.org/abs/math-ph/0702079
Ich glaube nicht, dass dich jemand angeschrien oder dir Namen genannt hat, sicherlich haben die Physiker den höchsten Respekt vor allem, was du schreibst. Dazu gehört auch, dass du dir über mehrere Jahre hinweg den Kopf gebrochen hast, um jede einzelne deiner Ideen zu verinnerlichen, auch die, die es nicht sind Nicht 100% richtig. Die Leute sagten nur, Sie hätten nicht 100% Recht, weil die No-Gos nicht umgangen werden. Das ist ein bisschen stumpf, aber nicht wirklich unhöflich. Ich glaube, ich habe eine etwas andere Art gefunden, das zu tun, was Sie wollen. Ich werde es als Antwort veröffentlichen.
Lieber @QuestionAnswers,, Sie interpretieren falsch, was ich sage: Ich glaube nicht, dass diese Fragen illegal sind. Es ist legal, sie zu fragen, sie wurden vor ungefähr 90 Jahren gefragt und sie wurden vor 85 Jahren beantwortet. Es ist albern und nicht illegal, sie 2012 erneut zu fragen, weil die Physik die Antwort schon seit einiger Zeit kennt. Es ist eine ziemlich lange Zeit. 85 Jahre nachdem die Physiker festgestellt hatten, dass Heliozentrismus richtiger als Geozentrismus ist, wurde es allgemein als dumm angesehen, den Heliozentrismus erneut in Frage zu stellen. Lernen in der Moderne sollte schneller sein, ist es aber anscheinend nicht.
@LubošMotl: Dies ist eine Fehlcharakterisierung. Sie können es als mathematische Übung betrachten. Ich möchte QM ungefähr simulieren, aber mein Computer ist zu klein, um den Status zu speichern. Kann ich dies mit einem Computer tun, dessen Größe ungefähr (groß) konstant mal der Anzahl der Partikel wächst? Ist es möglich? Dies ist im Wesentlichen das, was t'Hooft verlangt. Es wird von den No-Go-Theoremen nicht ausgeschlossen, wenn es ausreichend nicht lokal ist (das Beispiel, das ich gebe, ist in M-Kopien, die alle miteinander interagieren, schrecklich nicht lokal). Die Aussage, dass es unmöglich ist, QM aus versteckten Variablen zu reproduzieren, wird von Bohm ausdrücklich widerlegt.
Lieber @Ron,, Sie können offensichtlich keine Quantenphänomene mit einem klassischen Computer simulieren, dessen Größe als Anzahl der Teilchen skaliert. Wenn dies möglich wäre, könnte man einfache klassische Algorithmen aufschreiben, um die schnellen Algorithmen nachzuahmen, die nur auf Quanten-PCs funktionieren. Sie kennen diese Dinge sicher, nicht wahr? Warum versuchen Sie, diese Frage als eine gute zu verkaufen? Nichtlokalität ist kein Rezept, um die Quantenmechanik zu emulieren. Realismus statt Lokalität ist hier die falsche Annahme. Nonl. realistische Theorien können auch gefälscht werden, siehe z. http://motls.blogspot.cz/2007/04/falsifying-quantum-realism-again.html?m=1
Um Quantenfreiheitsgrade zu simulieren, müssen Sie sich eindeutig an die gesamte Wellenfunktion erinnern und sie als "klassisches beobachtbares Objekt, das zusammenbricht" behandeln, damit die Komplexität exponentiell mit der Anzahl der Quantenfreiheitsgrade zunimmt. Trotz dieser exponentiellen Investition erhalten Sie nicht die richtige * Theorie * der physikalischen Phänomene. Sie erhalten nur eine Simulation, die zur Computerspielbranche gehört, nicht zur Wissenschaft. In der Wissenschaft geht es um die wirklichen Phänomene, nicht um die besten Möglichkeiten, sie vorzutäuschen. Die Computerimitation müsste unendlich fein abgestimmt werden, um grundlegende Funktionen wie Lorentz sym.
@LubošMotl: Ja, natürlich können Sie keine Quantenberechnung simulieren. Die Sache ist, diese Arten von Quantencomputerzuständen sind unglaublich verwickelt und sehr schwer zu realisieren, ohne dass Dekohärenz sie verdirbt, so sehr, dass wir solche Zustände experimentell nicht realisiert haben. Die Frage ist, ob Sie mit einem linear skalierenden Computer das tägliche QM, viel Dekohärenz und keinen Quantencomputer simulieren können. Man könnte sagen "kollabieren", aber das ist schwieriger als es rechnerisch aussieht. In einem diskreten QM-Analogon erhalten Sie einen automatischen Zusammenbruch, und Sie können immer Monte-Carlo ausführen.
@QuestionAnswers: Obwohl ich Ihren Standpunkt sehe, ist es nicht gut, in der Philosophie voreingenommen zu sein. Lubos 'Interpretation von "akausalen Ereignissen" unterscheidet sich nicht so sehr von vielen Welten, sie unterscheiden sich nur durch Hokuspokus, und viele Welten sind oberflächlich realistisch (obwohl man das Modell von t'Hooft in vielen Welten betrachtet: Die universelle Wellenfunktion wird niemals überlagert, aber eine sieht, dass die Projektionen auf relative Zustände relative Zustände definitiv überlagern - wie passiert das wirklich? Es gibt keine Projektion, es ist schwieriger zu sehen als in üblichen QM-Modellen).
@LubošMotl: Der Satz, den Sie verwendet haben, um gegen den nichtlokalen Realismus zu argumentieren, ist nicht gut - er verwendet heimlich Variationen der Lokalität, um das Argument zu vertreten, nämlich indem Annahmen über die Art der statistischen Gemische getroffen werden, die eine realistische Theorie haben darf. In der Beschreibung, die ich gegeben habe, können Sie sehen, wie schrecklich die Nichtlokalität diese Art von Annahmen auf natürliche Weise verderben kann. Außerdem behaupten Sie in Ihrem Blog, dass Feldtheorie und Relativitätstheorie lokal und daher Physik lokal sind, aber Sie wissen sehr gut, dass die Stringtheorie nicht lokal ist und die Holographie die Lokalität aus dem Fenster geworfen hat.
Sehr geehrte @Ron,, ich denke nicht, dass Ihre Frage zur Simulation "ohne viel Verstrickung" in irgendeiner Weise klar definiert ist. Je klassischer eine Situation ist, desto weniger wichtig werden die Quantenphänomene. Aber an Verstrickungen ist nichts auszusetzen. Verschränkung ist die * Norm * in der Quantenmechanik. Wenn man sich mit charakteristischen Phänomenen der Quantenmechanik befasst, ist eine nahezu maximale Verschränkung die Regel, keine Ausnahme, und hier wird die Unzulänglichkeit eines "klassischen Modells" am offensichtlichsten. Man kann höchstens versuchen, seinen Kopf im Sand zu verstecken und die Fälschung zu übersehen.
Lieber @QuestionAnswers,, Sie haben geschrieben: "Die Sache ist, für SIE wurde die Antwort vor 80 Jahren gegeben. Aber für die meisten Realisten war es nicht, daher Interpretation vieler Welten, De-Broglie-Böhm-Interpretation, Zick-Zack-Interpretation in der Zeit und so weiter." Nein, so funktioniert Wissenschaft nicht. Beweise in der Wissenschaft sind nicht subjektiv. Die Beweise wurden vor 85 - nicht nur vor 80 - Jahren gefunden und waren für alle da. Die Beweise zeigten irreversibel, dass jede "realistische Beschreibung" mit Beobachtungen unvereinbar ist. Jeder, der sich als "Realist unabhängig von Beweisen" deklariert, wird garantiert für immer falsch liegen.
Sehr geehrte @Ron,, die 2007 von Zeilinger et al. In diesem Artikel werden Klassen von Modellen, die nicht lokal sind, explizit erörtert, und es kann immer noch gezeigt werden, dass sie mit den Beobachtungen nicht kompatibel sind, was beweist, dass Nichtlokalität hier nicht die "Heilung" ist. Sie versuchen nur, Nebel auf diese unbestreitbaren und etablierten Tatsachen zu werfen. In jedem Fall ist diese Diskussion akademisch, da seit 1905 festgestellt wurde, dass die Naturgesetze in der flachen Raumzeit genau lokal sind; Die Lokalität ergibt sich aus der Lorentz-Symmetrie. Sie sind einfach nicht klassisch (d. H. "Realistisch"). Die Natur folgt quanten, d. H. Unrealistischen, probabilistischen Gesetzen.
@LubošMotl: Sie wiederholen lächerliche Propaganda, als ob es eine Tatsache wäre, das ist nur bedauerliche Angstmacherei. Zeilingers "Klassen von Modellen" ist eine sehr kurzsichtige Klasse, die weder meine noch ein anderes vernünftiges nichtlokales Modell enthält. Lokalität und Lorentz-Invarianz haben nichts miteinander zu tun, ungeachtet der naiven Argumente. Hier ist eine Lorentz-invariante nichtlokale Aktion: $ \ int {\ phi (x) \ phi (y) \ over ((xy) ^ 2 + 1) ^ {. 73}} d ^ 4x d ^ 4y $, es gibt viele von Anderen. Die Lokalität fehlt in der Stringtheorie, es gibt keine lokalen Felder und sie fehlt vollständig in der AdS / CFT-Masse, in der die gesamte Raumzeit entsteht.
... in Bezug auf "Realismus" wird der lokale Realismus mit Bell'scher Ungleichung und der "kleine Realismus", der unter Verwendung von Shors Algorithmus das exakte QM reproduziert, widerlegt. Alle anderen Einschränkungen sind schwächer. Böhms Theorie zeigt, dass man den (exponentiell großen) Realismus nicht widerlegen kann, weil er funktioniert und realistisch ist, also gibt es kein allgemeines No-Go. Ein moderner "Realist" sucht jedoch nicht nach Böhms Theorie, sondern nach einer Theorie, die das QM im stark verschränkten Bereich nicht reproduziert und daher tatsächlich unterschiedliche Vorhersagen liefert. Eine solche Theorie ist das, wonach t'Hooft strebt, und es ist sinnvoll, danach zu suchen.
... selbst wenn es physikalisch falsch ist, wenn es mathematisch richtig ist, ist es ein Rezept zur Reduzierung stark verwickelter Zustände. Verstrickung ist natürlich die Norm! Aber wir nennen es normalerweise "Zusammenbruch", nicht Verschränkung, und es geht normalerweise nur in eine Richtung - die Reduzierung der Komplexität eines Quantensystems. Die heiklen Fälle der Quantenberechnung erfordern eine Feinabstimmung, damit die Verschränkung hin und her geht und nicht kollabiert, sondern nicht trivial berechnet wird. In den meisten Fällen können Sie die Verschränkung als Zusammenbruch annähern. Eine "realistische Annäherung" an die Quantenmechanik ist nützlich für die automatische Kollaps-Simulation.
Lieber @Ron,, Ihre Handlung ist Lorentz-invariant, aber akausal: Ein Effekt kann seine Vergangenheit beeinflussen. Wenn also nicht gezeigt werden kann, dass ähnliche feldtheoretische Aktionen einer lokalen Aktion entsprechen, erzeugen sie eine inkonsistente Theorie. Es ist nicht wirklich wahr, dass man auch in der Stringtheorie ähnliche "Nichtlokalitäten" bekommt. Wenn Sie die Freiheitsgrade korrekt als Funktionen der Massenschwerpunkte des Massenschwerpunkts ausdrücken, ist die Dynamik genau lokal, siehe z. http://arxiv.org/abs/hep-th/0406199 - Was ich oben geschrieben habe, ist keine "Propaganda", sondern Grundlagen der Quantenmechanik.
Auch hier ist es nicht wahr, dass nur der lokale Realismus als Grundlage für eine Theorie der Phänomene in der mikroskopischen Welt widerlegt wird. Realismus als solcher ist ausgeschlossen und die Gründerväter der Quantenmechanik kennen diese Tatsache seit Mitte der 1920er Jahre. Es gibt keinen Zusammenbruch in der realen Welt, und selbst wenn Sie die reale Welt nach einem klassischen Modell imitieren - und wieder geht es in der Physik um die Natur, nicht um Imitationen, und man kann Standardargumente der Physik verwenden, um zu zeigen, dass die Imitation nicht die reale ist Deal - der Zusammenbruch ist nicht dasselbe wie Verstrickung. Verschränkung muss auch in einer Nachahmung "generisch" sein.
@QuestionAnswers: "Sind Realisten ich?" - Ja, ich würde auf dieser Website eine andere Terminologie verwenden, aber das sind sie. Ja, ich lasse dich ein i sein. Aber es ist immer noch meine Pflicht in diesem Forum, darauf hinzuweisen, dass die "Realisten" ich sind. Wenn Sie verhindern möchten, dass Menschen sagen, dass die Physik seit 85 Jahren weiß, dass die Grundgesetze quantenhaft, dh nicht realistisch sind, müssen Sie versuchen, ein modernes Gegenstück der Inquisition zu kontaktieren, aber bereit sein, dass solche Institutionen weniger mächtig sind als sie war zu Galileos Zeiten.
Wie @RonMaimon erklärt hat, wurde der Realismus offensichtlich nicht widerlegt. Ich verstehe nicht einmal, wie man behaupten kann, Wissenschaft zu betreiben, wenn man versucht, die Realität selbst loszuwerden. Es ist mir jedoch auch egal. Ich bin mir genauso sicher, dass Sie falsch liegen, wie Sie sicher sind, dass Sie Recht haben.
Das eigentliche Problem, von dem ich denke, dass es von Ron Maimon und einigen anderen in ihren früheren Beiträgen am deutlichsten aufgedeckt wurde: Das Problem ist, dass in einer CA die "ontische" Wellenfunktion des Universums nur in bestimmten Modi der CA sein kann. Dies bedeutet, dass sich das Universum nur in Zuständen psi_1, psi_2, ... befinden kann, die die Voreigenschaft (psi_i | psi_j) = delta_ij (Entschuldigung für diese Nicht-Latex-Notation) haben, während die Quantenwelt, die wir beschreiben möchten, dies zulässt viel mehr Zustände, die überhaupt nicht orthonormal zueinander sind. Wie könnten diese Zustände jemals entstehen?
Ich bin versucht zu glauben, dass die Antwort auf diese Frage radikal ist, was viele von Ihnen verärgern könnte: - | Um unsere Welt zu beschreiben, haben wir den Hilbert-Raum erfunden, der nicht nur Basiselemente, sondern auch alle Überlagerungen enthält. - | wir haben gelernt zu denken, dass dieser Hilbert-Raum trennbar ist, das heißt, in jedem infinitesimalen Volue-Element dieser Welt gibt es einen solchen Hilbert-Raum, und der gesamte Hibert-Raum ist das Produkt all dieser. - | Normalerweise nehmen wir an, dass jeder der Zustände in diesem gemeinsamen Hilbert-Raum einen "ontischen" Zustand des Universums darstellen kann.
- | Das ärgerliche ist, dass dies möglicherweise nicht wahr ist. Die ontischen Zustände des Universums können eine viel kleinere Klasse von Zuständen bilden psi_i- | Alles, was wir annehmen müssen, ist, dass alle ontischen Zustände des Universums eine orthonormale Menge bilden. Diese orthonormale Menge ist NICHT trennbar, und deshalb denken wir lokal, dass wir nicht nur die Basiselemente, sondern auch alle Überlagerungen haben. Beachten Sie, dass solche Sätze leicht vorstellbar sind. Diese orthonormale Menge lässt sich dann leicht auf einen Automaten abbilden. Es besteht kein Grund zu der Annahme, dass dieser Automat kein lokaler Automat sein kann.
Um genau zu sein: Die ontischen Zustände sind nur trennbar, wenn sie mit den CA-Zuständen als Basis dargestellt werden. Die Zustände sind nicht trennbar, wenn sie mit Standardmodellzuständen (SM) als Basis beschrieben werden. Die SM-Zustände gehorchen lokalen Diff-Gleichungen, wenn sie als CA-Zustände ausgedrückt werden, aber die Lösungen dieser Gleichungen sind nicht lokal.
Haben Sie ein einfaches CA-Modell, das eine Überauswahl aufweist?
@LubošMotl: Erstens bedeutet das Gross / Erler-Papier nicht, dass die Stringtheorie "lokal" ist, sondern "Lichtkegel-lokal", was nicht genau dasselbe ist. Die Lichtkegellokalität der String-Wechselwirkung wurde erstmals in den 1970er Jahren von Mandelstam gezeigt, im Wesentlichen, um eine kausale Anfangswertformulierung zu erhalten, und dies war der Grund, warum die String-Feldtheorie überhaupt formuliert wurde. Wenn die Lichtkegelkoordinaten dunkel sind, wie wenn Sie einen Gravitationshorizont haben, werden die Argumente nicht übersetzt, weshalb die Stringfeldtheorie nicht so grundlegend ist, sie ist nicht vollständig störungsfrei.
@LubošMotl: Zweitens sind Lorentz-Invarianz und -Lokalität getrennte Konzepte, und dies gilt auch für Theorien, die eine Anfangswertformulierung zulassen. Sie können eine Aktion $ \ int \ phi (x) \ phi (y) G (xy) $ verwenden, bei der G (xy) im vorderen Lichtkegel nur ungleich Null ist, und einen Phasenraum erstellen, der die gesamte Vergangenheit von enthält das klassische Feld $ \ phi $. Diese dummen Trickaktionen sind keine guten Aktionen, aber sie sind ausgeschlossen, weil sie nicht korrekt quantenmäßig sind, nicht weil sie nicht lokal und Lorentz-invariant sind. Die AdS / CFT-Konstruktion in der Stringtheorie (oder Matrixtheorie) ist eindeutig nicht lokal.
@LubošMotl: In Bezug auf die Realität neige ich dazu, die Quantenmechanik für genau zu halten, weil ich Interpretationen ohne Kollaps als philosophisch konsistent in sich akzeptiere. Aber ich akzeptiere nicht, dass Realismus tot ist, weil Böhm real und Böhm kausal ist und Böhm gleichbedeutend mit QM ist. Es ist nicht gut als Theorie, aber es ist gut als Gegenbeispiel zu übermäßig starken Behauptungen.
@G.'tHooft: Ich bin verwirrt - haben Sie etwas falsch mit der Konstruktion gefunden, die ich in der Antwort vorgeschlagen habe? Es ist nicht das, was du tust, aber es ist für mich zu diesem Zweck so natürlich, dass ich lange dachte, es sei das, was du tust, und ich war verwirrt, als ich nicht abbilden konnte, was du mit dieser Sache gemacht hast. Ich bin mir ziemlich sicher, dass es wirklich funktioniert, einen orthogonalen Hilbert-Raum in einen Wahrscheinlichkeitsraum einzubetten, und danach ist es ziemlich einfach, sich auf die Einheitlichkeit zu beschränken. Meinen Sie in Bezug auf die Nichttrennbarkeit eine nicht unzählige Anzahl von Automatenzuständen? Dies ist eine unnötige Kürzung.
Als Experimentator bin ich mit den Argumenten des Universums verloren. Es scheint mir alles über mathematische Intuition und Beweise oder No-Go-Theoreme zu sein. Warum kann diese berühmte CA nicht mit einem Proton beginnen, sagen wir, oder noch einfacher mit einem Elektron? Der Geist ist verwirrt, wenn man das gesamte Universum braucht, um eine Elektronenstreuung an einem Proton zu untersuchen
@annav: Man muss ein ganzes klassisches Universum untersuchen (viel mehr Daten, als man normalerweise zwei Teilchen zuordnen würde), um auch nur ein Quantenproton zu beschreiben, das von einem Quantenelektronen gestreut wird, weil man die Quantenmechanik nicht auf kleine Weise entstehen lassen kann - - Die Berechnung im Multi-Partikel-QM ist einfach zu groß. Die verschiedenen Teile des klassischen Universums schnüffeln an verschiedenen Quantenoptionen, um das QM im Kleinen zu reproduzieren. Dies ist so nicht lokal, dass es kaum vorstellbar ist, und nur aufgrund des holographischen Prinzips - wir wissen, dass das Proton und das Elektron sowieso verschmiert sind.
Entschuldigung für die Verzögerung, aber ich habe die Frage zu $ ​​\ delta \ rho $ nicht beantwortet, weil ich keine nichttrivialen Quantensysteme konstruiert hatte (ich habe das infinitesimale Problem nur am Mittwoch behoben). Ich denke, ich kann Böhm reproduzieren. Die beabsichtigte Interpretation ist, dass $ \ delta \ rho $ die Wellenfunktion ist. Wenn ich Böhm von einer Grenze aus reproduziere, werde ich alles wissen. Es sieht aus wie ein Abschluss von Bohm, so dass die Wellenfunktion eine Funktion der Positionen der Instanziierungen ist (es gibt viele). Ich werde die Details ausfüllen, wenn ich mit der Ausarbeitung fertig bin (ich habe einige an diesem Wochenende gemacht, aber ich habe nie genug Zeit).
Es ist nichts, was "jeder schon weiß". Wie aus all diesen Threads hervorgeht, kennen Sie oder Gerard 't Hooft diese Dinge offensichtlich nicht, abgesehen von etwa 95% der Benutzer, die diese Threads besuchen. Vielleicht merken Sie beide wirklich nicht, dass Sie QM ablehnen, aber das liegt daran, dass Sie es völlig falsch verstehen. In Wirklichkeit leugnen Sie jedes einzelne Postulat der Quantenmechanik. Dies gilt für die Annahmen, Zwischenergebnisse sowie Anwendungen von QM. Zum Beispiel ist es logisch unmöglich, dass eine "Theorie, die QM reproduziert" impliziert, dass Quantencomputer nicht funktionieren.
@ Motl: Anscheinend axiomatisieren Sie QM, indem Sie es auf "Postulate" stützen. Natürlich werden Sie meine Theorie nicht verstehen, wenn Sie nicht bereit sind, Änderungen vorzunehmen, da Ihre Postulate ungenau sind. Sie sagten, dass "Experimente gezeigt haben, dass man Quantenzustände überlagern kann". Nicht wahr, Sie können dies nur mit den Vorlagen tun, die Sie verwenden, aber nicht in der realen Welt. Wenn Sie die Überlagerung zweier Zustände betrachten, ignorieren Sie die Umgebung dieser beiden Zustände, die niemals gleich und daher immer orthogonal sind.
@ Motl: In gewöhnlichen QM-Anwendungen können Sie dies ignorieren, da die Vorlagen gut genug sind, aber nicht in Fragen der Interpretation von QM.
@'t Hooft, ich bin sehr gespannt, warum Sie böhmische Mechaniker verabscheuen. Es scheint mir, dass es bereits das getan hat, was Sie versuchen - eine vollkommen konsistente realistische Alternative zur Quantenmechanik zu bieten.
@user7348, Meinst du diese Pilotwellen? Ich denke, das sind hässliche Erfindungen, aber ich stimme zu, dass sie zeigen, dass es im Prinzip eine Möglichkeit gibt. Ich denke, dass Eleganz und Plausibilität wichtige Werte einer gesunden Theorie sein werden. Ich kann mit Bohm keine Arbeitsfeldtheorien aufstellen. Ich spreche von viel grundlegenderen Prinzipien. Und vor allem: Meine Theorie ist Quantenmechanik, keine "Alternative".
@ 't Hooft: Warum kannst du mit Bohm keine Arbeitsfeldtheorie machen? Dies liegt daran, dass es sehr schwierig ist, die Theorie mit der Relativitätstheorie vereinbar zu machen. Wenn Sie über Grundprinzipien sprechen möchten, haben Sie meines Erachtens das gesamte Grundprinzip von Bell's Theorem übersehen - dass die Natur nicht lokal ist. Ich bin mir nicht sicher, ob Sie jemals EPR gelesen haben, aber Einstein argumentierte, dass die Quantenmechanik unvollständig sein müsse, um das zu vermeiden, was er berühmt als "gruselige Fernwirkung" bezeichnete. Bell zeigte, dass Einsteins Idee nicht funktionierte, sagte er, "es ist eine Schande, dass es nicht funktioniert". Was lässt uns das übrig? Gruslig!
@user7348 Ich appelliere zwar nie an die Behörde, möchte aber nur darauf hinweisen, dass G. 't Hooft ein Nobelpreisträger ist. Wenn Sie seine Anmeldeinformationen überprüfen möchten, gehen Sie einfach zu Wikipedia. Natürlich ist er sich des Glockensatzes, des EPR und dessen bewusst, was der Satz UNTER BESTIMMTEN ANNAHMEN zeigt. Hooft stellt die Annahme in Frage, dass Sie "frei" sind, X anstelle von Y unter genau denselben Anfangsbedingungen zu messen. Eine bekannte Lücke, von der Bell selbst schon einige Male erzählt hat. Er nannte es "Superdeterminismus".
@QuestionAnswers: Und ich bin mir der Superdeterminismus-Schleife sehr wohl bewusst. Bell schlug dies als Möglichkeit vor, aber er tat dies als intellektuelle Neugier, die alle möglichen Wege erkundete. Er glaubte es nicht wirklich. Ich verstehe, dass Hooft einer der wichtigsten Physiker des letzten halben Jahrhunderts ist, aber ich habe das Gefühl, dass er "seinen Kopf in den Sand steckt", wie Bell von allen außer Einstein sagte. Bell und seine Arbeit sind wahrscheinlich die am meisten missverstandenen in der Geschichte der Physik. Die Natur ist nicht lokal, das ist Bell's Theorem, Einsteins Programm ist gescheitert.
@QuestionAnswers / 't Hooft: Ich denke, es ist an der Zeit, über das eigentliche Problem zu spekulieren: Wie kann Nichtlokalität mit Relativitätstheorie vereinbar gemacht werden?
@user7348 beweisen es. Wie beinhaltet die Vielwelteninterpretation Nichtlokalität? Wie genau brechen Hoofts Modelle zusammen, wenn er die Annahme, Glockensicherheit zu konstruieren, nicht akzeptiert?
@QuestionAnswers: Ich kann es nicht beweisen. Aber viele Welten gehören in die Kurbelakte, und Superdeterminismus ist eine Verschwörung. Ich würde alles tun, um Einsteins Reaktion auf Bells Theorem zu hören.
@user7348 Erklären Sie, wie MWI ankurbelt und warum der von 't Hooft vorgeschlagene Superdeterminismus falsch ist. Wenn nicht, hören Sie bitte auf, an Diskussionen über' t Hoofts-Theorien teilzunehmen. Alles, was Sie wirklich tun, ist zu sagen, dass der Glockensatz richtig ist.
@QuestionAnswers: Fair genug. Ich habe Hooft nie beleidigt. Ich wollte nur verstehen, warum er Nicht-Lokalität nicht mag. Für mich ist es selbstverständlich, es anzunehmen und nicht zu leugnen. Aber fair genug.
Superdeterminismus ist offensichtlich da, wenn Sie darüber nachdenken möchten. Wenn Leute von "Verschwörung" sprechen, meinen sie wirklich, dass sie das Ergebnis nicht verstehen. Aber Sie können es so verstehen: Die CA kann als ausgewachsenes Quantensystem behandelt werden, als QFT. Diese QFT führt zu Korrelationen, die wie Verschwörung aussehen. Daher ist diese offensichtliche Verschwörung da. Hab keine Angst vor Gespenstern. Mathematisch ist nichts falsch an ihnen, nur ein bisschen schwierig.
Sie verletzen sogar nicht die Lokalität, da die QFT Kommutatoren hat, die immer außerhalb des Lichtkegels verschwinden. Es könnte sein, dass das Mapping eine rudimentäre Nichtlokalität aufweist, aber ich bin dem noch nicht wirklich begegnet.
OK, lassen Sie mich diese letzte Aussage korrigieren. Einige Pseudo-Nicht-Lokalitäten können auf zwei Arten eingegeben werden: 1: die Beschreibung des Vakuumzustands als Überlagerung von CA-Zuständen. Das Vakuum macht die Diskussion über Bell's Ungleichungen in CA sehr schwierig. 2: Es gibt gute Gründe, sich einen Informationsverlust in der Zertifizierungsstelle vorzustellen. Sie können dies immer noch auf ein Quantensystem abbilden (mit vollständiger CPT-Invarianz), aber diese Abbildung führt zu Holographie und * offensichtlicher * Nichtlokalität.
@ 't Hooft: Es würde mich wirklich interessieren zu hören, was Sie zu der folgenden Frage zu sagen haben, da ich den höchsten Respekt für Ihre Leistungen habe. Lassen Sie uns für einen Moment Ihre jüngsten Arbeiten beiseite legen und den Satz von Bell diskutieren, wie er in den 1960er Jahren diskutiert werden konnte, als Bell seine Entdeckung machte. Wenn man die Quantenmechanik so nimmt, wie sie ist, ohne zusätzliche versteckte Parameter, ohne Modifikationen, können wir dann sagen, dass die Quantenmechanik selbst nicht lokal ist? Ich frage, weil das DIE Kritik Einsteins an der Quantenmechanik zu sein scheint.
@'t Hooft: Ich möchte mich auch bei Ihnen für die Teilnahme an physics.stackexchange.com bedanken. Es war faszinierend, Gespräche mit einem wahren Riesen der Physik zu führen.
@user 7348: 1: Wenn Sie sich nicht für versteckte Variablen interessieren, ist die Quantenmechanik wie sie ist, genauer gesagt die Quantenfeldtheorie, vollständig lokal. Lokalität bedeutet, dass wenn wir in der Heisenberg-Notation zwei Feldoperatoren haben, die von der Raumzeit abhängen: $ Op_1 (x_1, t_1) $ und $ Op_2 (x_2, t_2) $, dann müssen sie pendeln, wenn $ (x_1, t_1) $ und $ (x_2, t_2) $ sind vollständig raumartig getrennt. Dies gilt für QFT und sogar (trotz gegenteiliger Behauptungen) für die Stringtheorie (wenn zwei Punkte im Zielraum raumartig getrennt sind, sind sie dies auch im Weltblatt - vorausgesetzt, wir können bestimmte Projektionen ignorieren).
Für viele Physiker ist dies alles, was zählt, auch in den 1960er Jahren. Ich nehme an, Einstein hat an so etwas wie versteckte Variablen gedacht. 2: MIT versteckten Variablen behauptet Bell, dass die versteckten Variablen nicht lokal sind, aber was er wirklich meint ist, dass die Ansatz-Gleichung, mit der er beginnt, nicht erfüllt werden kann. Meine Behauptung ist, dass in einem superdeterministischen Szenario diese Gleichung nicht gelten kann, selbst wenn die Evolutionsgesetze einer CA lokal sind.
't Hooft, ich möchte keine jugendliche Frage stellen, aber wie können Sie in einem Experiment vom Typ Verschränkung erklären, wie die Drehungen immer korrelieren, wenn keine versteckten Variablen vorhanden sind? Es muss eine gewisse Kommunikation stattfinden, ich denke, das war der Punkt von EPR - um zu zeigen, dass die Quantenmechanik durch ein reductio ad absurdum-Argument unvollständig ist. Aber wenn Sie den Satz von Bell außer Acht lassen, zeigt dies, dass Einstein falsch lag. Daher stimme ich nicht zu, dass "wenn wir uns nicht um versteckte Variablen kümmern, die Quantenmechanik, wie sie ist, vollständig lokal ist".
@user7348 Siehe meine Antwort unter http://physics.stackexchange.com/questions/34650/definitions-locality-vs-causality/34675#34675. In verschränkten Zuständen findet keine Kommunikation statt, es gibt nur frühere Korrelationen.
@user7348 Was man in einer Quantentheorie benötigen muss, um lokal oder kausal zu sein, ist, dass * Observable * in raumähnlicher Entfernung anstelle von Feldern pendeln. Es gibt Theorien oder Formulierungen von Theorien, in denen die Felder nicht in raumähnlichen Entfernungen pendeln, sondern vollkommen lokal oder kausal sind, da dies bei den Observablen der Fall ist. Ich Gast, Hooft würde nicht zustimmen.
@ Drake: Ich weiß nicht, über welche Theorien du sprichst. In meiner Theorie, die aus einer lokalen Kartierung von einer lokalen Zertifizierungsstelle erhalten wurde, liegt die einzige Nichtlokalität über einer kleinen Anzahl von Gitterplätzen. Weiter entfernt verschwinden alle Kommutatoren außerhalb des Lichtkegels.
@ user7348: Ich akzeptiere den Satz von Bell nicht so einfach. Es ist * schwierig *, genau zu sehen, was passiert, aber es ist entscheidend, dass alle CA-Observablen jederzeit pendeln. Nach meiner einheitlichen Zuordnung werden daher nur Observablen, die orthogonal zueinander sind, in Bezug auf CA-Variablen eindeutig definiert. Da die Zuordnung kompliziert ist, unterscheiden sich diese Observablen jedes Mal, wenn Sie ein Experiment durchführen. Daher können wir * kontrafaktische * Observablen haben, die nicht pendeln.
Wenn Sie ein Experiment wiederholen oder es viele Male durchführen, können Sie daher ein beobachtbares Experiment nicht ändern, ohne irgendwo ein anderes zu beeinflussen. Es ist schwer zu verstehen, wie dies geschieht. Sie müssen sich daran erinnern, dass das Vakuum, das uns umgibt, ein sehr komplizierter verwickelter Zustand ist. All dies ist der wahre Grund, warum Bell in der CA verletzt werden kann. Also ignoriere ich Bell und in diesem Fall ist qm (eher: qft) lokal.
@'t Hooft, es tut mir leid, aber Ihre Antwort scheint nicht zum Thema zu gehören. Ich habe nicht nach Ihrer CA-Theorie gefragt. Ich sagte, es fällt mir schwer zu verstehen, wie Korrelationen in der Standardquantenmechanik erklärt werden können, ohne dass eine "gruselige" Kommunikation stattfindet. Ich bin in einer Situation, in der ich, obwohl ich bereit bin, den Realismus aufzugeben, immer noch denke, dass es ein Problem mit der Relativität gibt. Meine Frage an Sie lautet also: Angenommen, Sie sind wie ich und können den Realismus aufgeben und die Quantenmechanik vollständig akzeptieren. Haben wir in der Ferne gruselige Aktionen? Nennen Sie das eine dumme Frage, aber es ist Einsteins EPR-Frage.
@user7348: Nein, in der Quantenfeldtheorie ist die Kausalität eingebaut. dafür genügt es zu verlangen, dass alle Kommutatoren außerhalb des Lichtkegels verschwinden. Dies garantiert, dass kein Signal jemals schneller als Licht geht. Qft gehorcht also der Relativitätstheorie und hat gewöhnliche qm-Teilchen in ihrer nicht-relativistischen Grenze. Alles ist in Ordnung, kein Problem mit der Relativitätstheorie, bis Sie versuchen zu verstehen, was die Ontologie ist. Sie müssen sich daran erinnern, dass raumartige Korrelationen in Ordnung sind, wenn Sie sie in der Vergangenheit mit Anfangszuständen erklären können.
Stellen Sie sich qft als einen großen Satz von Quantenharmonischen Oszillatoren vor, die jeweils an isolierten Punkten im Raum schwingen. Nehmen wir dann an, dass jeder Oszillator nur Interaktionen mit seinen direkten Nachbarn zeigt. In qft sind dies * Quanten * -Interaktionen. Für die meisten Theoretiker sieht dies ausreichend lokal aus, keine gruseligen Signale.
@ 't Hooft: Danke' t Hooft! Vielen Dank für Ihre Antworten und vielen Dank für Ihre Teilnahme an physics.stackexchange!
@ 't Hooft: Ich habe tatsächlich eine andere Frage, eine, die sehr gut für einen Meister der QFT geeignet ist. Sie haben erwähnt, dass die böhmische Mechanik nicht zur Konstruktion von Feldtheorien verwendet werden kann. Ich habe diese Überraschung immer gefunden, da sie die Quantenmechanik reproduziert. Warum kann eine böhmische Feldtheorie nicht funktionieren?
Im Prinzip sollte man ja in der Lage sein, eine böhmische Feldtheorie zu konstruieren, aber ich denke, das wäre unelegant. Nach meinem Geschmack fügt die böhmische Mechanik viel zu viele "nicht beobachtbare Observable" in Form von Pilotwellen hinzu. Dies wäre schrecklich für Feldtheorien, bei denen die Pilotwelle eine Feldfunktion oder eine Funktion von unendlich vielen Teilchenpositionen wäre.
Sechs antworten:
Peter Shor
2012-08-17 19:11:43 UTC
view on stackexchange narkive permalink

Ich kann Ihnen sagen, warum ich nicht daran glaube. Ich denke, meine Gründe unterscheiden sich jedoch von den Gründen der meisten Physiker.

Regelmäßige Quantenmechanik impliziert die Existenz von Quantenberechnungen. Wenn Sie an die Schwierigkeit des Factorings (und eine Reihe anderer klassischer Probleme) glauben, dann scheint eine deterministische Grundlage für die Quantenmechanik eine der folgenden zu implizieren.

  • Es gibt einen klassischen Polynom-Zeit-Algorithmus für Factoring und andere Probleme, die auf einem Quantencomputer gelöst werden können.
  • Die deterministischen Grundlagen der Quantenmechanik erfordern $ 2 ^ n $ Ressourcen für ein System der Größe $ O (n) $.
  • Die Quantenberechnung funktioniert in der Praxis nicht.

Keines davon scheint mir überhaupt wahrscheinlich . Zum einen ist es durchaus denkbar, dass es einen Polynom-Zeit-Algorithmus zum Faktorisieren gibt, aber die Quantenberechnung kann viele ähnliche Periodizitätsprobleme lösen, und Sie können argumentieren, dass es keinen einzigen Algorithmus geben kann, der alle auf einem löst klassischer Computer, daher müssten Sie für jedes klassische Problem unterschiedliche klassische Algorithmen haben, die ein Quantencomputer durch Periodenfindung lösen kann.

Für die zweite, deterministische Grundlage der Quantenmechanik, für die $ 2 ^ n $ Ressourcen erforderlich sind Ein System der Größe $ O (n) $ ist wirklich unbefriedigend (aber vielleicht durchaus möglich ... schließlich fällt die Theorie, dass das Universum eine Simulation auf einem klassischen Computer ist, in diese Klasse von Theorien und kann, obwohl es wirklich unbefriedigend ist, ' durch dieses Argument nicht ausgeschlossen werden).

Zum dritten habe ich keinen vernünftigen Weg gefunden, wie Sie die Quantenberechnung unmöglich machen können, während Sie die Konsistenz mit den aktuellen experimentellen Ergebnissen beibehalten.

Ich gebe zu, dass meine Stärke heutzutage die Informatik und nicht die Physik ist, aber würde keine deterministische QM-Theorie implizieren, dass die Laufzeit eines Algorithmus auf einem Quantencomputer auch eine Fiktion ist?Denken Sie daran, dass die Qualitätskontrolle die gesamte Geschichte des Universums hatte, um seinen internen Zustand vorzubereiten, damit programmierte Probleme in subklassischer Zeit gelöst werden konnten.Die Qualitätskontrolle führt im Grunde eine klassische, teilweise evaluierte Version Ihrer Berechnung aus, bei der ein Großteil der Arbeit bereits erledigt wurde, bevor Sie auf "Ausführen" geklickt haben.
@naasking: Wenn Sie einen $ 2 ^ n $ -Algorithmus für mäßig große $ n $ ausführen müssen, ist es nicht so hilfreich, die gesamte Geschichte des Universums zu haben.
Können Sie erklären, warum Determinismus => Quantencomputer nicht funktioniert?Die böhmische Mechanik entspricht experimentell der traditionellen Quantenmechanik *. Wie kann es sein, dass eine deterministische Theorie das Quantencomputing ausschließt?* mit Ausnahme kleinerer Einschränkungen, die die Fähigkeit, N verschränkte 2-Ebenen-Systeme oder Logikgatter herzustellen, nicht beeinträchtigen
@doublefelix;Bohmian Mechanics ist nicht lokal, was bedeutet, dass ein System der Größe $ O (n) $ meines Erachtens keine feste Größenbeschreibung hat.Dies ist die zweite Möglichkeit, die ich in meiner Antwort aufführe.Meine Antwort erlaubt also böhmische Mechanik.Und wenn Sie sich die Frage ansehen, sagt Hooft nicht: "Also bin ich zu einem reversiblen, lokalen, deterministischen Automaten zurückgekehrt und habe gefragt: Inwieweit ähnelt dies dem QM und wo geht es schief?"Die Frage schließt also die böhmische Mechanik aus, da die böhmische Mechanik nicht lokal ist.
Christoph
2012-08-20 23:59:28 UTC
view on stackexchange narkive permalink

Dies hätte ein Kommentar sein können, aber da er die im Titel gestellte Frage tatsächlich beantwortet, werde ich ihn als solchen posten:

Soweit ich das beurteilen kann, gibt es keinen vernünftigen Grund, diese zu verwerfen Modelle außer Kontrolle - es ist nur so, dass die Quantenmechanik (QM) die Messlatte furchtbar hoch gelegt hat: Bisher gibt es keine experimentellen Beweise dafür, dass QM falsch ist, und niemand hat eine praktikable Alternative gefunden.

Letztendlich muss Ihre Theorie alle experimentell verifizierten Vorhersagen von QM reproduzieren (oder kann nur innerhalb der experimentellen Genauigkeit abweichen). Es ist jedoch natürlich nicht erforderlich, willkürliche Vorhersagen zu reproduzieren. Wenn Sie dies tatsächlich tun, erhalten Sie eine Neuformulierung - dh eine neue Interpretation - des normalen QM. Wenn Ihr Modell uns sagt, dass eine groß angelegte Quantenberechnung unmöglich ist, liegt es an den Experimentatoren, Ihnen das Gegenteil zu beweisen.

Alle darüber hinausgehenden Einwände sind nur Psychologie am Werk: Die meisten Menschen müssen sich ziemlich viel Mühe geben Überzeugen Sie sich selbst, dass QM eine gültige Beschreibung der Welt ist, in der wir leben, und sobald ein solcher Glaube tief verwurzelt ist, wird er leicht zum Dogma.

Arnold Neumaier
2012-08-15 15:32:51 UTC
view on stackexchange narkive permalink

Grundlegende Diskussionen ähneln in der Tat Diskussionen über religiöse Überzeugungen, da man Annahmen und Ansätze auf der Ebene der Grundlagen nicht beweisen oder widerlegen kann.

Darüber hinaus liegt es in der Natur der Diskussionen im Internet, dass man wahrscheinlich Antworten hauptsächlich von denen erhält, die entweder stark anderer Meinung sind (der Fall hier) oder die etwas Konstruktives hinzufügen können (was sehr schwierig ist) aktuelle Forschung). Ich denke, dies erklärt die Antworten, die Sie erhalten, vollständig.

Ich selbst habe einen Ihrer Artikel dazu oberflächlich gelesen und fand ihn nicht vielversprechend genug, um mehr Zeit mit technischen Fragen zu verbringen.

Ich stimme jedoch zu, dass sowohl Vielwelten als auch Pilotwellen inakzeptable physikalische Erklärungen der Quantenphysik sind, und ich arbeite an einer alternativen Interpretation.

Meiner Ansicht nach wird die Nichtlokalität von Partikeln dadurch erklärt, dass Partikel jede ontologische Existenz negieren. Es gibt Quantenfelder, und auf der Quantenfeldebene ist alles lokal. Nichtlokale Merkmale treten nur auf, wenn man den Feldern eine Teilcheninterpretation auferlegt, die zwar unter den üblichen Annahmen der geometrischen Optik gültig ist, aber bei höherer Auflösung drastisch versagt. Daher muss im Bereich des Scheiterns nichts erklärt werden. So wie die lokalen Maxwell-Gleichungen für ein klassisches elektromagnetisches Feld die Nichtlokalität einzelner Photonen erklären (Doppelspaltexperimente) und die stochastischen Maxwell-Gleichungen alles über einzelne Photonen erklären (siehe http://www.mat.univie.ac.at/). ~ neum / ms / optslides.pdf), daher erklärt die lokale QFT die allgemeine Nichtlokalität von Partikeln.

Meine thermische Interpretation der Quantenphysik (siehe http://www.mat.univie.ac.at/~neum/physfaq/therm) gibt einen Überblick über die Physik im Einklang mit der tatsächlichen experimentellen Praxis und ohne die Seltsamkeit, die durch die üblichen Interpretationen eingeführt wird. Ich glaube, dass diese Interpretation in jeder Hinsicht zufriedenstellend ist, obwohl es mehr Zeit und Mühe erfordert, die Standardprobleme in dieser Richtung zu analysieren, wobei eine klare Ableitung der statistischen Mechanik meine bisher hauptsächlich qualitativen Argumente stützt.

In Ich hatte ähnliche Schwierigkeiten wie Sie, als ich meine grundlegenden Ansichten in Online-Diskussionen präsentierte. siehe z. B. den PhysicsForums-Thread "Was behauptet die probabilistische Interpretation von QM?" http://www.physicsforums.com/showthread.php?t=480072

Das Problem ist, dass Ihre Interpretation Standard-QM ist und ob Sie Felder oder Partikel als Grundvariablen verwenden, Sie immer noch die exponentielle Explosion haben. Die Feldwellenfunktion ist genauso schrecklich wie die Teilchenwellenfunktion. Das Modell, das t'Hooft möchte (und ich möchte auch), würde keine exponentiellen Ressourcen für ein großes Raster benötigen. Sie können die Philosophie nicht einfach neu ausrichten, um dies zu erreichen, indem Sie den "ontischen" Status von Objekten in der Theorie ändern (ehrlich gesagt kümmert sich ein Positivist nicht einmal um den ontischen Status).
@RonMaimon: Interpretationen der Quantenphysik haben per se nichts mit Rechenkomplexität zu tun, sondern damit, aus den Standardquantenphänomenen einen gemeinsamen Sinn zu machen.
Ja, normalerweise ist dies wahr, und deshalb ist diese Frage nicht gewöhnlich. Der Punkt der Untersuchung von 'tHooft besteht darin, eine neue Theorie (nicht die Quantenmechanik) zu formulieren, die dennoch annähernd quantenmechanisch ist, aber nicht unter der exponentiellen Explosion der rechnerischen Komplexität leidet, die das gewöhnliche QM plagt, und sich daher experimentell vom gewöhnlichen QM unterscheidet für den Fall eines Quantencomputers. Das Ziel einer solchen Theorie ist es, in Fällen, in denen Sie mit den vorliegenden Experimenten nicht einverstanden sind, experimentell nicht anders zu sein. Das ist eine schwierige Sache.
@RonMaimon: Während dies in seinem Modell der Fall sein mag, ist es nicht sein ausdrückliches Ziel. Stattdessen schrieb er oben: '' Ich kann nicht anders, als von der Interpretation "vieler Welten" oder den "Pilotwellen" von Bohm - deBroglie angewidert zu sein, und selbst die Idee, dass die Quantenwelt nicht lokal sein muss, ist schwer zu kaufen . Ich möchte wissen, was wirklich los ist, und um einige Ideen zu bekommen, konstruiere ich einige Modelle mit unterschiedlichem Grad an Raffinesse. ''
Ja, ich weiß, die angegebenen Motivationen laden zu vielen philosophischen Antworten ein, aber dies ist wirklich nicht die Motivation. Ich weiß das, weil ich das holographische Prinzip verstehe, die Zeitungen lese und vor zehn Jahren einmal kurz mit ihm gesprochen habe. Die Motivation besteht darin, kleine versteckte Variablen zu erstellen, dh eine Anzahl von Bits, die bei holographischen Einschränkungen eine klassische Berechnung von angemessener Größe ermöglichen und die QM ungefähr reproduzieren. Es ist nicht QM genau zu reproduzieren. Aber er macht einen Fehler in seinen Papieren und bekommt ein zu gutes QM, also denkt er, dass es es lokal und fast genau reproduziert.
Zu sagen, dass Felder das grundlegende Konzept sind und dass sich Felder lokal verhalten, umgeht nicht die Tatsache, dass Sie entweder Nichtlokalität oder exponentielle Ressourcen benötigen, um eine Quantenberechnung zu ermöglichen.Daher ist Ihr Programm zum Finden einer lokalen feldbasierten Erklärung von QFT wahrscheinlich zum Scheitern verurteilt.
@PeterShor: Die thermische Interpretation der Quantenphysik hat trotz der Lokalität der Operatorfeldgleichungen tatsächlich einen teilweise nichtlokalen Charakter, da die (potenziell messbaren) dynamischen Entitäten, die eine geschlossene Dynamik erfüllen, die nichtlokalen n-Punkt-Korrelationsfunktionen sind$ n> 1 $.
Ron Maimon
2012-08-16 10:37:13 UTC
view on stackexchange narkive permalink

Hier gibt es zwei Fragen: Warum Ihre Modelle kritisieren? Und gibt es bessere Ideen? Ich werde versuchen, die zweite Frage in einer separaten Antwort zu beantworten. Hier gebe ich nur einige allgemeine Kommentare, um die erste Frage anzusprechen.

Ich stimme Ihnen persönlich zu, und ich denke, die meisten Leute, die sich für dieses Zeug interessieren, tun es auch, dass es beunruhigend ist, eine Theorie zu haben wobei die durch Beobachtungen erzeugten Informationen nicht in der Theorie selbst enthalten sind, sondern durch einen Messvorgang aus der Luft erzeugt werden. Die natürliche Idee ist, dass, wenn wir ein Stück Information sehen, das durch einen Beobachtungsakt erzeugt wurde, der Wert dieses Bits irgendwie in der vollständigen Beschreibung der Natur enthalten war, unabhängig vom Beobachtungsakt. Dies war Einsteins Realitätsprinzip, und ich stimme zu, dass es für eine Theorie bevorzugt ist, ihm zu gehorchen.

Wenn eine Theorie dem Realitätsprinzip nicht gehorcht, muss man beachten, dass die makroskopische Realität ihr gehorcht, und die Teile in der makroskopischen Welt durch einen philosophisch verzerrten Kreisverkehr finden, der in Mystik trainiert. Aber da die Physik empirisch und der Positivismus fruchtbar ist, bin ich der Ansicht, dass jeder Rahmen, der die Ergebnisse von Beobachtungen erklärt, letztendlich philosophisch in Ordnung sein muss, selbst wenn er Verzerrungen erfordert und selbst wenn er nicht korrekt ist! Obwohl Newtons Mechanik, obwohl sie falsch ist, nicht notwendigerweise empirisch widerlegt wird, wenn nur Beobachtungen von Menschen usw. gemacht werden, darf sie nicht philosophisch mit dem freien Willen unvereinbar sein. In ähnlicher Weise mag die Quantenmechanik falsch sein, aber wir haben keine empirischen Daten, die zeigen, dass sie falsch ist. Daher sollte es philosophisch konsistent sein zu sagen, dass QM alles ist, was es gibt. Dies bedeutet, dass QM auch Beobachter beschreiben sollte, und wenn es keinen mathematischen Widerspruch zu dieser Ansicht gibt, sollte es auch keinen philosophischen Widerspruch geben, selbst wenn es einen Widerspruch zum Experiment gibt. Dies ist die Philosophie vieler Welten, und es ist die selbstkonsistente Antwort, wenn die Quantenmechanik korrekt ist. Es mag ärgerlich sein, aber ich denke nicht, dass es zu ärgerlich ist - man sollte einfach lernen, mit vielen Welten als einer guten philosophischen Position zu leben.

Aber es ist falsch, einfach "viele" zu sagen -welten "an diesem Punkt, weil die Quantenbeschreibung nicht in dem Bereich getestet wurde, in dem die vielen Welten eine echte logisch-positivistische Manifestation haben - am offensichtlichsten, wenn enorm große Zahlen mit berücksichtigt werden ein Quantencomputer. Bis wir dies tun, ist es definitiv denkbar, dass die Natur für kleine Systeme mit wenigen Teilchen nur sehr nahe am Quanten liegt, in den Fällen, in denen wir die Theorie bereits getestet haben, und für stark verschränkte Teilchensysteme einfach kein Quanten ist

Selbst wenn sich herausstellt, dass die Welt wirklich quantenmäßig ist und ein Quantencomputer ständig Zahlen enthält, ist es hilfreich, eine deterministische Substruktur zu finden, um in Fällen, in denen nicht ein Quantencomputer, und es ist möglich, dass diese Kürzung für Quantensimulationen nützlich sein kann. Dies ist so notwendig, dass ich denke, eine Unterstruktur für die Quantenmechanik zu finden, ist ein zentrales wichtiges Problem, unabhängig davon, ob es sich als richtig herausstellt. Aus diesem Grund habe ich viel Zeit darauf verwendet, Ihren Ansatz zu verstehen.

Das Problem bei Ihrer Konstruktion ist, dass sie zu gut funktioniert und es zu einfach ist, ein Quantensystem in ein Quantensystem umzuwandeln Beable-Basis, so dass sich die globale Wellenfunktion auf eine Weise entwickelt, die unter Verwendung des globalen Hamilton-Operators deterministisch ist. Da Sie den Hilbert-Raum frühzeitig einführen und damit die Transformation der Basis in die internen Zustände des Automaten durchführen, gibt es weder eine offensichtliche Barriere für die Umwandlung eines Quantencomputers in eine Beable-Basis noch eine Barriere für die lokale Verletzung der Bellschen Ungleichung. Diese deuten nicht darauf hin, dass die No-Go-Theoreme fehlerhaft sind, sondern darauf, dass die Transformation auf eine Beable-Basis mit einer Permutation nach Hamilton kein echtes klassisches System erzeugt.

Die genaue Art und Weise, wie ich glaube, dass dieses System nicht klassisch ist, liegt in der staatlichen Vorbereitung im Inneren. Der Prozess der Zustandsvorbereitung umfasst eine Messung, bei der ein inneres Teilsystem mit einem makroskopischen Teilsystem verschränkt wird, und anschließend eine Reduktion des makroskopischen Systems gemäß der Bornschen Regel, wobei ein reiner Quantenzustand des inneren Teilsystems verbleibt. In Ihrer Arbeit über Borns Regel haben Sie vorgeschlagen, wie die Reduktion in einem CA-System erfolgen soll, aber Ihre genauen Modelle respektieren diese Intuition nicht wirklich, da die Messung von Zwischenzuständen immer einen der Eigenzustände des im Inneren beobachtbaren erzeugt. egal wie kompliziert das Beobachtbare ist und wie verwickelt seine Eigenzustände sind. Dies ermöglicht es Ihnen, die Quantenmechanik auf den inneren Subsystemen zu reproduzieren. Ich bin mir ziemlich sicher, dass dies den Zustand in der Beable-Basis nicht unüberlagert hält. Da diese internen Reduktionen die Wahrscheinlichkeitsstruktur nicht berücksichtigen, machen Sie wirklich Quantenmechanik, nicht CA, und dies ist der einzige Grund, warum es Ihnen so leicht fällt, die No-Gos zu umgehen.

Die Tatsache, dass Wenn Sie das No-Gos ohne Schwierigkeiten umgehen, deutet dies stark darauf hin, dass Ihre Konstruktion den Raum zulässiger klassischer Wahrscheinlichkeitsverteilungen auf der CA irgendwie verlässt. Der einzige Ort, an dem dies geschehen kann, ist die interne Zustandsvorbereitung, die Messung interner Bediener. So bereiten Sie schließlich Bell-Zustände oder Quantencomputer vor. Diese inneren Operationen müssen Zustände (nach der Projektion) erzeugen, die nicht als klassische Wahrscheinlichkeitszustände des Automaten interpretiert werden können, obwohl die Hamiltonsche Evolution dies niemals tut. Dies ist kein Beweis, aber ich würde viel Geld wetten (wenn ich welche hätte). Ich habe hier um einen Beweis gebeten: Halten Messungen in 't Hooft Beable-Modellen Zustände klassisch?

Dies ist Teil I der Antwort, ich poste es separat, damit Leute, die diesem Teil zustimmen, den zweiten Teil, der einem anderen Ansatz gewidmet ist, um die Quantenmechanik aus Automaten herauszuholen, nicht zustimmen müssen Beantworten Sie die zweite Frage.

Ron Maimon
2012-08-16 11:45:23 UTC
view on stackexchange narkive permalink

Diese Frage versucht, die Quantenmechanik aus klassischen Automaten mit einem wahrscheinlich unbekannten Zustand zu reproduzieren.

Wahrscheinlichkeitsverteilungen auf Automatenzuständen

Beginnen Sie mit einer klassischen CA und einer Wahrscheinlichkeitsverteilung auf der CA . Um die Dinge allgemein zu halten, erlaube ich der CA eine nicht deterministische Evolution, aber nur eine stochastische Wahrscheinlichkeit, keine Quantenentwicklung, und es ist nicht notwendig, dass Sie die Wahrscheinlichkeit immer in die Anfangsbedingungen setzen können, ohne Stochastizität zu Zwischenzeiten Nur eine Option.

Der erste Punkt zu diesen stochastischen Systemen wird hier detailliert beschrieben: Konsequenzen des neuen Satzes in QM? (im Abschnitt über Entenfüße). Befindet sich der Wahrscheinlichkeitsfluss immer zwischen Zuständen, in denen sich die Wahrscheinlichkeit nur geringfügig von der stationären Verteilung unterscheidet, so bewahrt der klassische Fluss die Entropie und ist reversibel, selbst wenn er probabilistisch und diffus ist. Dies ist die zentrale Motivation für die Konstruktion, und man sollte überprüfen, wie das im Wärmetauscher-Diffusor diffundierende Partikel reversibel von Raum zu Raum hin und her springt, und zwar auf lineare Weise, die von einem Bediener mit einem meist komplexen Eigenwert beschrieben wird, obwohl dies der Fall ist diffundiert zu jeder Zeit nur zwischen verschiedenen erlaubten Regionen.

Betrachten Sie eine klassische Wahrscheinlichkeitsverteilung auf einer CA, $ \ rho (B) $, wobei B der Zustand aller Bits ist, aus denen der Automat besteht, dann

$$ I = - \ sum_B \ rho (B) \ log (\ rho (B)) $$

ist die Information, die enthalten ist, um den Automatenzustand vollständig zu kennen, über dem von der Vertrieb. Wenn Sie die erste Ordnung stören und $ \ rho $ in $ \ rho + \ delta \ rho $ ändern, finden Sie

$$ I = - \ sum_B (\ rho (B) + \ delta \ rho (B)) \ log (\ rho (B) + \ delta \ rho (B)) $$

Wenn $ \ rho $ einheitlich ist, verschwindet die Korrektur erster Ordnung seit der Summe von $ \ delta \ rho $ ist Null, und die Korrektur zweiter Ordnung ergibt eine quadratische metrische Struktur für $ \ delta \ rho $.

$$ I = - \ sum_B \ delta \ rho (B) ^ 2 $$

Dies ist das, was ich als Vorquantenstruktur im Raum der Störungen der Gleichverteilung identifiziere. Der Grund, warum es so symmetrisch ist (wie eine Kugel, nicht wie ein Simplex), ist, dass die Störung gering ist. Die Reversibilität ist für die Erhaltung der Entropie erforderlich, und die Erhaltung der Entropie erfordert, dass alle Transformationen auf $ \ delta \ rho $ orthogonal sind.

Das Bild zur nullten Ordnung ist, dass fast jeder Zustand gleich wahrscheinlich ist, aber Einige Zustände sind etwas wahrscheinlicher als andere, und die durch Experimente aufgedeckten Informationen führen nur für einige Zustände zu einer geringfügigen Verzerrung und nicht für andere. Diese geringfügigen Verzerrungen sind dann symmetrischer als der zugrunde liegende Wahrscheinlichkeitsraum für Automatenzustände, da diese Verteilungen nie genug von der Gleichmäßigkeit abweichen, um die Ecken des Wahrscheinlichkeitsraum-Simplex zu sehen. Die Ecken sind die Zustände, in denen die Automatenbits mit Sicherheit bekannt sind. Wenn Sie immer weit von diesen entfernt sind, können Sie eine symmetrische und reversible probabilistische Dynamik finden.

Hier ist das zentrale Problem bei diesem Ansatz: - Es ist unmöglich, dass eine Information, die die Störung $ \ delta \ rho $ enthält, überall klein ist. Der Grund ist, dass ein überall kleines $ \ delta \ rho $ notwendigerweise einen Zustand erzeugt, der vom einheitlichen Zustand fast nicht zu unterscheiden ist und daher eine Störung verursacht, die bedeutet, dass Sie viel weniger als nur 1 Bit Information gelernt haben. Wenn Sie beispielsweise einen N-Bit-Automaten haben und eine Verteilung erstellen, bei der die Wahrscheinlichkeit für jeden Bitwert zwischen $ {1 \ über 2} - \ epsilon $ und $ {1 \ über 2} + \ epsilon $ liegt, erhalten Sie Ein Informationsinhalt, der oben durch ein kleines Vielfaches von $ \ epsilon $ -Bits begrenzt ist.

Der Grund dafür ist, dass das Lernen von nur einem Bit Informationen über einen Automatenzustand die Anzahl der Zustände, die Sie einnehmen können, grob um den Faktor 2 verringert. Dies bedeutet, dass die wahre Wahrscheinlichkeitsverteilung erheblich sein muss klein bei mindestens der Hälfte der Konfigurationen, und es kann keine kleine Störung sein. Dies bedeutet, dass die Informationserweiterung zusammenbricht und ich hier lange Zeit feststeckte.

Lokal kleine Störungen

Der Grund, warum der Begriff "kleine Störung" fehlschlägt, ist, dass Eine kleine Störung, wie im Beispiel der Entenfüße, ist nicht global klein, sondern hat nur die Eigenschaft, dass das Verhältnis der Wahrscheinlichkeiten zwischen zwei nahe gelegenen Zuständen klein ist. Wenn die Zustände durch unabhängiges Variieren vieler Bits erstellt werden, gibt es viele Zustände mit demselben Wahrscheinlichkeitsverhältnis.

Die Korrektur könnte genauso gut der folgende einfache Trick sein: Erhöhen Sie einfach alles auf die M-te Potenz . Wenn Sie ein System mit durch i indizierten Zuständen haben, eine Ganzzahl im Bereich 1,2, ..., N und eine Störung

$$ (\ rho_i + \ delta \ rho_i) $$

Sie können die M-te Tensorleistung von $ \ rho $ verwenden, um eine Produktverteilung auf dem Tensorraum mit den M-Indizes $ i_1, i_2, ..., i_M $ zu erstellen. Diese Produktverteilung wird durch die Bedingung definiert, dass das Ändern jedes Wertes i von einem Wert zu einem anderen die gleiche Wahrscheinlichkeitsänderung des Verhältnisses bewirkt.

Jetzt darf $ \ delta \ rho $ klein sein, selbst wenn die Informationen in $ \ delta \ rho $ sind nicht vorhanden, da die M-te Potenz überhaupt nicht klein ist. Wenn Sie in diesem System wissen, dass der Informationsgehalt von $ \ rho + \ delta \ rho $ insgesamt 1 Bit beträgt, lernen Sie, dass

$$ M \ sum_B ist, da es sich um ein Tensorprodukt handelt \ delta \ rho ^ 2 = 1 $$

Mit anderen Worten, die endlichen Informationsstörungen der stationären Verteilung auf einem System mit M-Kopien bilden einen (realen, nicht komplexen) Hilbert-Raum, der immer perfekter wird, wenn M ins Unendliche geht. Wenn die Dynamik Entenfüße ist, was bedeutet, dass die Entropie mit der kleinen Störung erhalten bleibt, dann ist die zeitliche Entwicklung von $ \ delta \ rho $ notwendigerweise eine orthogonale Transformation, unabhängig vom zugrunde liegenden stochastischen oder deterministischen Evolutionsgesetz.

Die Grundidee ist, dass Sie eine Quantenmechanik aus der stochastischen Evolution von Systemen mit vielen identischen Kopien hervorbringen können, unter der Bedingung, dass die Kopien symmetrisch miteinander interagieren, sodass Sie nicht wissen, welche Kopie welche ist

Um zu sehen, wie das innere Produkt herauskommt, betrachten Sie die gegenseitigen Informationen, die Ihnen sagen, wie unabhängig zwei verschiedene Verteilungen sind. In der niedrigsten Reihenfolge wird dies gefunden, indem die Informationen in $ \ delta \ rho_1 $ und $ \ delta \ rho_2 $ genommen und die Informationen in $ \ delta \ rho_1 $ und $ \ delta \ rho_2 $ getrennt subtrahiert werden. Da dies die Normen sind, finden Sie

$$ I_ {12} = || \ delta \ rho_1 + \ delta \ rho_2 || ^ 2 - || \ delta \ rho_1 || ^ 2 - || \ delta \ rho_2 || ^ 2 = \ langle \ rho_1, \ rho_2 \ rangle $$

Wenn Sie also zwei Verteilungen haben, teilen sie sich die Zustände mit der Grad, in dem ihr inneres Produkt ungleich Null ist.

Kennen Sie dieses Dokument: http://arxiv.org/pdf/1111.6597.pdf und wenn ja, ist es auf Ihre Art von Konstruktion anwendbar?
@user1247: Diese Einschränkung ist relativ schwach, da davon ausgegangen wird, dass entfernte Systeme von unabhängigen Ensembles beschrieben werden (dies ist wiederum ein Ort, der sich einschleicht). Es wurde hier diskutiert: http://physics.stackexchange.com/questions/17170/consequences-of-the-new-theorem-in-qm. Ich habe es gelesen und verstanden und fand es interessant, aber es gilt nicht für diese Art von Dingen, wie Sie explizit sehen können, wenn Sie oben verschränkte Zustände konstruieren: Sie teilen immer das statistische Ensemble, egal wie weit sie voneinander entfernt sind sie sind, ihre Beschreibung ist nicht durch Verkettung jedes einzelnen.
Scary Monster
2012-08-18 14:01:14 UTC
view on stackexchange narkive permalink

Es besteht kein Zweifel, dass es möglich ist, quantenintegrierbare Modelle mit klassischen Systemen ziemlich effizient und einfach zu reproduzieren. Von allen integrierbaren Systemen gehören harmonische Oszillatoren zu den einfachsten. Die eigentliche Herausforderung besteht darin, nichtintegrierbare Quantensysteme zu reproduzieren. Können Sie das Quantenchaos reproduzieren? Können Sie quantenunintegrierbare Spinmodelle über ein 1d-Raumgitter reproduzieren? Der Versuch einer Störungstheorie anhand eines integrierbaren Modells stößt auf das Problem, dass die Anzahl der Feynman-Diagramme exponentiell mit der Anzahl der Schleifen zunimmt.

Es ist nur der Anfang in der Zukunft, den er möglicherweise kann.
Es gibt Zweifel! Es ist nicht trivial, einfache Quantenmechanik aus zellulären Automaten zu reproduzieren, und ich glaube nicht, dass 'hOoft es tut (obwohl ich denke, dass er dem sehr nahe gekommen ist und intuitiv genau richtig ist).
@Scary Monster:Die Behauptung lautet, dass * jede * Zertifizierungsstelle in der Sprache des QM umgewandelt werden kann, obwohl die QM-Modelle, die Sie erhalten, in den meisten Fällen uninteressant sind. es wird Zustände geben, und sie werden Schrödinger-Gleichungen gehorchen. Jetzt sind viele CA-Modelle rechnerisch universell und daher sicherlich nicht integrierbar, und daher wird erwartet, dass die zugehörige QM-Theorie auch nicht trivial ist. Aber natürlich ist die Mathematik viel schwieriger; Es ist viel lehrreicher, nach Fällen zu suchen, in denen Sie (störende) Berechnungen durchführen können.


Diese Fragen und Antworten wurden automatisch aus der englischen Sprache übersetzt.Der ursprüngliche Inhalt ist auf stackexchange verfügbar. Wir danken ihm für die cc by-sa 3.0-Lizenz, unter der er vertrieben wird.
Loading...